K
Khách

Hãy nhập câu hỏi của bạn vào đây, nếu là tài khoản VIP, bạn sẽ được ưu tiên trả lời.

9 tháng 5 2018

Đáp án D

Ta có

10 tháng 5 2017

1) X=log1-log2+log2-log3+...+log99-log100

=log1-log100

=0-2

=-2

Đáp án C

2)X=-log3100=-log3102=-2log3(2.5)=-2log32-2log35=-2a-2b

Đáp án A

AH
Akai Haruma
Giáo viên
12 tháng 11 2018

Bài 1:

\(A=\log_380=\log_3(2^4.5)=\log_3(2^4)+\log_3(5)\)

\(=4\log_32+\log_35=4a+b\)

\(B=\log_3(37,5)=\log_3(2^{-1}.75)=\log_3(2^{-1}.3.5^2)\)

\(=\log_3(2^{-1})+\log_33+\log_3(5^2)=-\log_32+1+2\log_35\)

\(=-a+1+2b\)

AH
Akai Haruma
Giáo viên
12 tháng 11 2018

Bài 2:

\(\log_{30}8=\frac{\log 8}{\log 30}=\frac{\log (2^3)}{\log (10.3)}=\frac{3\log2}{\log 10+\log 3}\)

\(=\frac{3\log (\frac{10}{5})}{1+\log 3}=\frac{3(\log 10-\log 5)}{1+\log 3}=\frac{3(1-b)}{1+a}\)

HQ
Hà Quang Minh
Giáo viên
26 tháng 8 2023

\(x=log_34+log_94\\ =log_34+\dfrac{1}{2}log_34\\ =log_34+log_32\\ =log_38\\ \Leftrightarrow3^x=8\)

Chọn B.

HQ
Hà Quang Minh
Giáo viên
22 tháng 9 2023

\({a^{\frac{1}{2}}} = b \Leftrightarrow {\log _a}b = \frac{1}{2} \Leftrightarrow 2{\log _a}b = 1\)

Chọn B.

AH
Akai Haruma
Giáo viên
11 tháng 8 2017

Lời giải:

Sử dụng công thức \(\log_ab=\frac{\ln b}{\ln a}\)

\(\Rightarrow A=\frac{\ln 2}{\ln 3}.\frac{\ln 3}{\ln 4}.\frac{\ln 4}{\ln 5}....\frac{\ln 15}{\ln 16}\)

\(\Leftrightarrow A=\frac{\ln 2}{\ln 16}=\log_{16}2=\frac{1}{4}\)

Đáp án C.

11 tháng 5 2016

\(D=\frac{\log_2\left(2a^2\right)+\left(\log_2a\right)a^{\log_2\left(\log_2a+1\right)}+\frac{1}{2}\log^2_2a^4}{\log_2a^3\left(3\log_2a+1\right)+1}=\frac{1+2\log_2a+\log_2a\left(\log_2a+1\right)+8\log^2_2a}{3\log_2a.\left(3\log_2a+1\right)+1}\)

    \(=\frac{9\log^2_2a+3\log_2a+1}{9\log^2_2a+3\log_2a+1}=1\)

\(log_a\left(a^3b^2\right)=log_aa^3+log_ab^2=3+2\cdot log_ab\)

=>B

QT
Quoc Tran Anh Le
Giáo viên
22 tháng 9 2023

a)     \({3^{{x^2} - 4x + 5}} = 9 \Leftrightarrow {x^2} - 4x + 5 = 2 \Leftrightarrow {x^2} - 4x + 3 = 0 \Leftrightarrow \left( {x - 3} \right)\left( {x - 1} \right) = 0\)

\( \Leftrightarrow \left[ \begin{array}{l}x = 3\\x = 1\end{array} \right.\)

Vậy phương trình có nghiệm là \(x \in \left\{ {1;3} \right\}\)

b)    \(0,{5^{2x - 4}} = 4 \Leftrightarrow 2x - 4 = {\log _{0,5}}4 \Leftrightarrow 2x = 2 \Leftrightarrow x = 1\)

Vậy phương trình có nghiệm là x = 1

c)     \({\log _3}(2x - 1) = 3\)    ĐK: \(2x - 1 > 0 \Leftrightarrow x > \frac{1}{2}\)

\( \Leftrightarrow 2x - 1 = 27 \Leftrightarrow x = 14\) (TMĐK)

Vậy phương trình có nghiệm là x = 14

d)    \(\log x + \log (x - 3) = 1\)  ĐK: \(x - 3 > 0 \Leftrightarrow x > 3\)

\(\begin{array}{l} \Leftrightarrow \log \left( {x.\left( {x - 3} \right)} \right) = 1\\ \Leftrightarrow {x^2} - 3x = 10\\ \Leftrightarrow {x^2} - 3x - 10 = 0\\ \Leftrightarrow \left( {x + 2} \right)\left( {x - 5} \right) = 0\\ \Leftrightarrow \left[ \begin{array}{l}x =  - 2 (loại) \,\,\,\\x = 5 (TMĐK) \,\,\,\,\,\,\,\end{array} \right.\end{array}\)

Vậy phương trình có nghiệm x = 5